Question 501 of 560

A 23 year old man presents with delayed diagnosis of appendicitis. The appendix is retrocaecal and has perforated causing a psoas abscess. Into which structure does the psoas major muscle insert?

Greater trochanter of the femur

Linea aspera of the femur

Lesser trochanter of the femur

Iliac crest

None of the above

Theme based on 2011 exam The psoas major inserts into the lesser trochanter. Please rate this question:

Discuss and give feedback Next question Psoas Muscle

Origin The deep part originates from the transverse processes of the five lumbar vertebrae, the superficial part originates from T12 and the first 4 lumbar vertebrae.

Insertion Lesser trochanter of the femur.

Innervation Anterior rami of L1 to L3.

Action Flexion and external rotation of the hip. Bilateral contraction can raise the trunk from the supine position. Next question

Question 502 of 560

A 63 year old man is due to undergo a splenectomy. Which splenic structure lies most posteriorly?

Gastrosplenic

Splenic vein

Splenic artery

Splenic notch

Lienorenal ligament

Theme from 2011 Exam The lienorenal ligament lies most posteriorly. The antero-lateral connection is via the . Anteriorly the gastro splenic ligament. These structures condense around the vessels at the splenic hilum.

Please rate this question:

Discuss and give feedback

Next question

Splenic anatomy

The spleen is the largest lymphoid organ in the body. It is an intraperitoneal organ, the peritoneal attachments condense at the hilum where the vessels enter the spleen. Its blood supply is from the splenic artery (derived from the coeliac axis) and the splenic vein (which is joined by the IMV and unites with the SMV).

 Embryology: derived from mesenchymal tissue  Shape: clenched fist  Position: below 9th-12th ribs  Weight: 75-150g

Relations

 Superiorly- diaphragm  Anteriorly- gastric impression  Posteriorly- kidney  Inferiorly- colon  Hilum: tail of pancreas and splenic vessels  Forms apex of lesser sac (containing short gastric vessels)

Next question

Question 503 of 560

Which of these statements relating to the external carotid is false?

It ends by bifurcating into the superficial temporal and ascending pharyngeal artery

Its first branch is the superior thyroid artery

The superior thyroid, lingual and facial arteries all arise from its anterior surface

The ascending pharyngeal artery is a medial branch

Initially it lies anteromedial to the internal carotid

It terminates by dividing into the superficial temporal and maxillary branches. The external carotid has eight branches, 3 from its anterior surface ; thyroid, lingual and facial. The pharyngeal artery is a medial branch. The posterior auricular and occipital are posterior branches.

Please rate this question:

Discuss and give feedback

Next question

External carotid artery

The external carotid commences immediately lateral to the pharyngeal side wall. It ascends and lies anterior to the internal carotid and posterior to the posterior belly of digastric and stylohyoid. More inferiorly it is covered by sternocleidomastoid, passed by hypoglossal nerves, lingual and facial veins. It then pierces the fascia of the parotid gland finally dividing into its terminal branches within the gland itself.

Surface marking of the carotid This is an imaginary line drawn from the bifurcation of the common carotid passing behind the angle of the jaw to a point immediately anterior to the tragus of the ear.

Branches of the external carotid artery It has six branches, three in front, two behind and one deep.

Three in front Superior thyroid Lingual Facial

Two behind Occipital Posterior auricular

Deep Ascending pharyngeal

It terminates by dividing into the superficial temporal and maxillary arteries in the parotid gland.

Image sourced from Wikipedia

Next question Question 504 of 560

Which of the following statements about the spleen is false?

The hilum contains the splenic vessels.

The spleen is derived from endodermal tissue.

The white pulp has immune function.

The colon lies inferiorly.

Weighs 150g.

1,3,5,7,9,11 (odd numbers up to 11)

The spleen is: 1 inch thick, 3 inches wide, 5 inches long, weighs 7oz (150-200g), lies between the 9th and 11th ribs

Most of the gut is derived endodermally except for the spleen which is from mesenchymal tissue. Please rate this question:

Discuss and give feedback Next question Spleen

The spleen is located in the left upper quadrant of the and its size can vary depending upon the amount of blood it contains. The typical adult spleen is 12.5cm long and 7.5cm wide. The usual weight of the adult spleen is 150g. The exact position of the spleen can vary with respiratory activity, posture and the state of surrounding viscera. It usually lies obliquely with its long axis aligned to the 9th, 10th and 11th ribs. It is separated from these ribs by both diaphragm and pleural cavity. The normal spleen is not palpable.

The shape of the spleen is influenced by the state of the colon and stomach. Gastric distension will cause the spleen to resemble the shape of an orange segment. Colonic distension will cause it to become more tetrahedral.

The spleen is almost entirely covered by , which adheres firmly to its capsule. Recesses of the greater sac separate it from the stomach and kidney. It develops from the upper dorsal mesogastrium, remaining connected to the posterior and stomach by two folds of peritoneum; the lienorenal ligament and . The lienorenal ligament is derived from peritoneum where the wall of the general peritoneum meets the omental bursa between the left kidney and spleen; the splenic vessels lie in its layers. The gastrosplenic ligament also has two layers, formed by the meeting of the walls of the greater sac and omental bursa between spleen and stomach, the short gastric and left gastroepiploic branches of the splenic artery pass in its layers. Laterally, the spleen is in contact with the phrenicocolic ligament.

Relations

Superiorly Diaphragm

Anteriorly Gastric impression

Posteriorly Kidney

Inferiorly Colon

Tail of pancreas and splenic vessels (splenic artery divides here, branches pass to the Hilum white pulp transporting plasma)

Contents

White Immune function. Contains central trabecular artery. The germinal centres are supplied pulp by arterioles called penicilliary radicles.

Red pulp Filters abnormal red blood cells.

Function

 Filtration of abnormal blood cells and foreign bodies such as bacteria.  Immunity: IgM. Production of properdin, and tuftsin which help target fungi and bacteria for phagocytosis.  Haematopoiesis: up to 5th month gestation or in haematological disorders.  Pooling: storage of 40% platelets.  Iron reutilisation  Storage monocytes

Disorders of the spleen Massive splenomegaly

 Myelofibrosis  Chronic myeloid leukaemia  Visceral leishmaniasis (kala-azar)  Malaria  Gaucher's syndrome

Other causes (as above plus)

 Portal hypertension e.g. secondary to cirrhosis  Lymphoproliferative disease e.g. CLL, Hodgkin's  Haemolytic anaemia  Infection: hepatitis, glandular fever  Infective endocarditis  Sickle-cell*, thalassaemia  Rheumatoid arthritis (Felty's syndrome)

*the majority of adult patients with sickle-cell will have an atrophied spleen due to repeated infarction Next question

Question 505 of 560

As it exits the axilla the radial nerve lies on which of the following muscles?

Supraspinatus

Infraspinatus

Teres major

Deltoid

Pectoralis major

The radial nerve passes through the triangular space to leave the axilla. The superior border of this is bounded by the teres major muscle to which the radial nerve is closely related. Please rate this question:

Discuss and give feedback Next question Radial nerve

Continuation of posterior cord of the brachial plexus (root values C5 to T1)

Path

 In the axilla: lies posterior to the axillary artery on subscapularis, latissimus dorsi and teres major.  Enters the arm between the brachial artery and the long head of triceps (medial to humerus).  Spirals around the posterior surface of the humerus in the groove for the radial nerve.  At the distal third of the lateral border of the humerus it then pierces the intermuscular septum and descends in front of the lateral epicondyle.  At the lateral epicondyle it lies deeply between brachialis and brachioradialis where it then divides into a superficial and deep terminal branch.  Deep branch crosses the supinator to become the posterior interosseous nerve.

In the image below the relationships of the radial nerve can be appreciated

Image sourced from Wikipedia

Regions innervated

 Triceps  Anconeus Motor (main nerve)  Brachioradialis  Extensor carpi radialis

 Supinator  Extensor carpi ulnaris  Extensor digitorum Motor (posterior  Extensor indicis interosseous branch)  Extensor digiti minimi  Extensor pollicis longus and brevis  Abductor pollicis longus

The area of skin supplying the proximal phalanges on the dorsal aspect of the Sensory hand is supplied by the radial nerve (this does not apply to the little finger and part of the ring finger)

Muscular innervation and effect of denervation Anatomical location Muscle affected Effect of paralysis Anatomical location Muscle affected Effect of paralysis

Shoulder Long head of triceps Minor effects on shoulder stability in abduction

Arm Triceps Loss of elbow extension

Forearm Supinator Weakening of supination of prone hand and Brachioradialis elbow flexion in mid prone position Extensor carpi radialis longus and brevis

The cutaneous sensation of the upper limb- illustrating the contribution of the radial nerve

Image sourced from Wikipedia

Next question

Question 506 of 560

Into which of the following veins does the middle thyroid vein drain?

Vertebral

External jugular

Internal jugular

Subclavian

Anterior jugular

It drains to the internal jugular vein. Which is one of the reasons why it bleeds so copiously if a ligature slips. Please rate this question:

Discuss and give feedback Next question Thyroid gland

 Right and left lobes connected by isthmus  Surrounded by sheath from pretracheal layer of deep fascia  Apex: Lamina of thyroid cartilage  Base: 4th-5th tracheal ring  Pyramidal lobe: from isthmus  May be attached to foramen caecum at the base of the tongue

Relations Anteromedially  Sternothyroid  Superior belly of omohyoid  Sternohyoid  Anterior aspect of sternocleidomastoid

Posterolaterally Carotid sheath Medially  Larynx  Trachea  Pharynx  Oesophagus  Cricothyroid muscle  External laryngeal nerve (near superior thyroid artery)  Recurrent laryngeal nerve (near inferior thyroid artery)

Posterior  Parathyroid glands  Anastomosis of superior and inferior thyroid arteries

Isthmus  Anteriorly: Sternothyroids, sternohyoids, anterior jugular veins  Posteriorly: 2nd, 3rd, 4th tracheal rings (attached via Ligament of Berry)

Blood Supply Arterial  Superior thyroid artery (1st branch of external carotid)  Inferior thyroid artery (from thyrocervical trunk)  Thyroidea ima (in 10% of population -from brachiocephalic artery or aorta)

Venous  Superior and middle thyroid veins - into the IJV  Inferior thyroid vein - into the brachiocephalic veins

Next question

Question 507 of 560

Which of the following structures is not at the level of the transpyloric plane?

Hilum left kidney

Superior mesenteric artery

Fundus of the gallbladder

Cardioesophageal junction

Root of transverse mesocolon

Cardiooesophageal junction level = T11

A knowledge of this anatomic level is commonly tested. The oesophagus extends from C6 (the lower border of the cricoid cartilage) to T11 at the cardioesophageal junction. Note that in the neonate the oesophagus extends from C4 or C5 to T9. Please rate this question:

Discuss and give feedback Next question Levels

Transpyloric plane Level of the body of L1

 Pylorus stomach  Left kidney hilum (L1- left one!)  Fundus of the gallbladder  Neck of pancreas  Duodenojejunal flexure  Superior mesenteric artery  Portal vein  Left and right colic flexure  Root of the transverse mesocolon  2nd part of the duodenum  Upper part of conus medullaris  Spleen

Can be identified by asking the supine patient to sit up without using their arms. The plane is located where the lateral border of the rectus muscle crosses the costal margin.

Anatomical planes Subcostal plane Lowest margin of 10th costal cartilage

Intercristal plane Level of body L4 (highest point of iliac crest)

Intertubercular plane Level of body L5

Common level landmarks Inferior mesenteric artery L3

Bifurcation of aorta into common iliac arteries L4

Formation of IVC L5 (union of common iliac veins)

Diaphragm apertures  Vena cava T8  Oesophagus T10  Aortic hiatus T12

Next question

Question 508 of 560

A 62 year old man presents with arm weakness. On examination he has a weakness of elbow extension and loss of sensation on the dorsal aspect of the first digit. What is the site of the most likely underlying defect?

Axillary nerve

Median nerve

Ulnar nerve

Radial nerve

Musculocutaneous nerve

Theme from April 2012 Exam The long head of the triceps muscle may be innervated by the axillary nerve and therefore complete loss of triceps muscles function may not be present even with proximally sited nerve lesions. Please rate this question:

Discuss and give feedback Next question Radial nerve

Continuation of posterior cord of the brachial plexus (root values C5 to T1)

Path

 In the axilla: lies posterior to the axillary artery on subscapularis, latissimus dorsi and teres major.  Enters the arm between the brachial artery and the long head of triceps (medial to humerus).  Spirals around the posterior surface of the humerus in the groove for the radial nerve.  At the distal third of the lateral border of the humerus it then pierces the intermuscular septum and descends in front of the lateral epicondyle.  At the lateral epicondyle it lies deeply between brachialis and brachioradialis where it then divides into a superficial and deep terminal branch.  Deep branch crosses the supinator to become the posterior interosseous nerve.

In the image below the relationships of the radial nerve can be appreciated

Image sourced from Wikipedia

Regions innervated

 Triceps  Anconeus Motor (main nerve)  Brachioradialis  Extensor carpi radialis

 Supinator  Extensor carpi ulnaris  Extensor digitorum Motor (posterior  Extensor indicis interosseous branch)  Extensor digiti minimi  Extensor pollicis longus and brevis  Abductor pollicis longus

The area of skin supplying the proximal phalanges on the dorsal aspect of the Sensory hand is supplied by the radial nerve (this does not apply to the little finger and part of the ring finger)

Muscular innervation and effect of denervation Anatomical location Muscle affected Effect of paralysis

Shoulder Long head of triceps Minor effects on shoulder stability in abduction

Arm Triceps Loss of elbow extension

Forearm Supinator Weakening of supination of prone hand and Brachioradialis elbow flexion in mid prone position Extensor carpi radialis longus and brevis

The cutaneous sensation of the upper limb- illustrating the contribution of the radial nerve

Image sourced from Wikipedia

Next question

Question 509 of 560

From which of the following structures does the long head of the triceps muscle arise?

Coracoid process

Acromion

Infraglenoid tubercle

Coraco-acromial ligament

Coraco-humeral ligament

The long head arises from the infraglenoid tubercle. The fleshy lateral and medial heads are attached to the posterior aspect of the humerus between the insertion of the teres minor and the olecranon fossa. Please rate this question:

Discuss and give feedback Next question Triceps

Origin  Long head- infraglenoid tubercle of the scapula.  Lateral head- dorsal surface of the humerus, lateral and proximal to the groove of the radial nerve  Medial head- posterior surface of the humerus on the inferomedial side of the radial groove and both of the intermuscular septae

Insertion  Olecranon process of the ulna. Here the olecranon bursa is between the triceps tendon and olecranon.  Some fibres insert to the deep fascia of the forearm, posterior capsule of the elbow (preventing the capsule from being trapped between olecranon and olecranon fossa during extension) Innervation Radial nerve

Blood Profunda brachii artery supply

Action Elbow extension. The long head can adduct the humerus and and extend it from a flexed position

Relations The radial nerve and profunda brachii vessels lie between the lateral and medial heads Next question

Question 510 of 560

A 45 year old man is undergoing a left hemicolectomy. As the surgeons mobilise the left colon they identify a tubular structure lying at the inferior aspect of psoas major. What is it most likely to be?

Left ureter

Left common iliac vein

Left common iliac artery

Left external iliac artery

Left external iliac vein

The left ureter lies posterior to the left colon. The sigmoid colon and upper rectum may be more closely related to the iliac vessels. These are not typically found above L4. Please rate this question:

Discuss and give feedback Next question Ureter

 25-35 cm long  Muscular tube lined by transitional epithelium  Surrounded by thick muscular coat. Becomes 3 muscular layers as it crosses the bony pelvis  Retroperitoneal structure overlying transverse processes L2-L5  Lies anterior to bifurcation of iliac vessels  Blood supply is segmental; renal artery, aortic branches, gonadal branches, common iliac and internal iliac  Lies beneath the uterine artery

Next question

Question 511 of 560

Which muscle is not innervated by the trigeminal nerve?

Medial pterygoid

Mylohyoid

Stylohyoid

Masseter

Temporalis

Stylohyoid is innervated by the facial nerve. Please rate this question:

Discuss and give feedback Next question Trigeminal nerve

The trigeminal nerve is the main sensory nerve of the head. In addition to its major sensory role, it also innervates the muscles of mastication.

Distribution of the trigeminal nerve Sensory  Scalp  Face  Oral cavity (and teeth)  Nose and sinuses  Dura mater

Motor  Muscles of mastication  Mylohyoid  Anterior belly of digastric  Tensor tympani  Tensor palati

Autonomic connections (ganglia)  Ciliary  Sphenopalatine  Otic  Submandibular

Path

 Originates at the pons  Sensory root forms the large, crescentic trigeminal ganglion within Meckel's cave, and contains the cell bodies of incoming sensory nerve fibres. Here the 3 branches exit.  The motor root cell bodies are in the pons and the motor fibres are distributed via the mandibular nerve. The motor root is not part of the trigeminal ganglion.

Branches of the trigeminal nerve Ophthalmic nerve Sensory only

Maxillary nerve Sensory only

Mandibular nerve Sensory and motor

Sensory Ophthalmic Exits skull via the superior orbital fissure Sensation of: scalp and forehead, the upper eyelid, the conjunctiva and cornea of the eye, the nose (including the tip of the nose, except alae nasi), the nasal mucosa, the frontal sinuses, and parts of the meninges (the dura and blood vessels).

Maxillary Exit skull via the foramen rotundum nerve Sensation: lower eyelid and cheek, the nares and upper lip, the upper teeth and gums, the nasal mucosa, the palate and roof of the pharynx, the maxillary, ethmoid and sphenoid sinuses, and parts of the meninges.

Mandibular Exit skull via the foramen ovale nerve Sensation: lower lip, the lower teeth and gums, the chin and jaw (except the angle of the jaw), parts of the external ear, and parts of the meninges.

Motor Distributed via the mandibular nerve. The following muscles of mastication are innervated:

 Masseter  Temporalis  Medial pterygoid  Lateral pterygoid

Other muscles innervated include:

 Tensor veli palatini  Mylohyoid  Anterior belly of digastric  Tensor tympani

Next question

Question 512 of 560

A 42 year old woman is due to undergo a left nephroureterectomy for a transitional cell carcinoma involving the ureter. Which of the following structures is not related to the left ureter?

Round ligament of the

Internal iliac artery

Ovarian artery

Peritoneum

Sigmoid mesocolon

The ureter is not related to the round ligament of the uterus, it is related to the broad ligament and is within 1.5cm of the supravaginal part of the cervix. Please rate this question:

Discuss and give feedback Next question Ureter

 25-35 cm long  Muscular tube lined by transitional epithelium  Surrounded by thick muscular coat. Becomes 3 muscular layers as it crosses the bony pelvis  Retroperitoneal structure overlying transverse processes L2-L5  Lies anterior to bifurcation of iliac vessels  Blood supply is segmental; renal artery, aortic branches, gonadal branches, common iliac and internal iliac  Lies beneath the uterine artery

Next question

Question 513 of 560

Which of the following most commonly arises from the brachiocephalic artery?

Vertebral artery

Subscapular artery

Thyroidea ima artery

Left Subclavian artery

None of the above

Other occasional branches include the thymic and bronchial branch. Please rate this question:

Discuss and give feedback Next question Brachiocephalic artery

The brachiocephalic artery is the largest branch of the aortic arch. From its aortic origin it ascends superiorly, it initially lies anterior to the trachea and then on its right hand side. It branches into the common carotid and right subclavian arteries at the level of the sternoclavicular joint.

Path Origin- apex of the midline of the aortic arch Passes superiorly and posteriorly to the right Divides into the right subclavian and right common carotid artery

Relations Anterior  Sternohyoid  Sternothyroid  Thymic remnants  Left brachiocephalic vein  Right inferior thyroid veins Posterior  Trachea  Right pleura

Right lateral  Right brachiocephalic vein  Superior part of SVC

Left lateral  Thymic remnants  Origin of left common carotid  Inferior thyroid veins  Trachea (higher level)

Branches Normally none but may have the thyroidea ima artery

Image sourced from Wikipedia

Next question

Question 514 of 560

A 28 year old man is undergoing an appendicectomy. The external oblique aponeurosis is incised and the underlying muscle split in the line of its fibres. At the medial edge of the wound is a tough fibrous structure. Entry to this structure will most likely encounter which of the following?

Internal oblique

Rectus abdominis

Transversus abdominis

Linea alba

Peritoneum

This structure will be the rectus sheath and when entered the rectus abdominis muscle will be encountered. Please rate this question:

Discuss and give feedback Next question Abdominal incisions

Midline incision  Commonest approach to the abdomen  Structures divided: linea alba, transversalis fascia, extraperitoneal fat, peritoneum (avoid above the umbilicus)  Bladder can be accessed via an extraperitoneal approach through the space of Retzius

Paramedian  Parallel to the midline (about 3-4cm) incision  Structures divided/retracted: anterior rectus sheath, rectus (retracted), posterior rectus sheath, transversalis fascia, extraperitoneal fat, peritoneum  Incision is closed in layers Battle  Similar location to paramedian but rectus displaced medially (and thus denervated)  Now seldom used

Kocher's Incision under right subcostal margin e.g. Cholecystectomy (open)

Lanz Incision in right iliac fossa e.g. Appendicectomy

Gridiron Oblique incision centered over McBurneys point- usually appendicectomy (less cosmetically acceptable than Lanz

Gable Rooftop incision

Pfannenstiel's Transverse supra pubic, primarily used to access pelvic organs

McEvedy's Groin incision e.g. Emergency repair strangulated femoral hernia

Rutherford Extraperitoneal approach to left or right lower quadrants. Gives excellent Morrison access to iliac vessels and is the approach of choice for first time renal transplantation.

Image sourced from Wikipedia

Next question

Question 515 of 560

A 35 year old man presents to the surgical clinic with a suspected direct inguinal hernia. These will pass through Hesselbach's triangle. Which of the following forms the medial edge of this structure?

External oblique aponeurosis

Inferior epigastric artery

Rectus abdominis muscle

Inferior epigastric vein

Obturator nerve

Direct inguinal hernias pass through Hesselbachs triangle (although this is of minimal clinical significance!). Its medial boundary is the rectus muscle.

Please rate this question:

Discuss and give feedback

Next question

Hesselbach's triangle

Direct hernias pass through Hesselbachs triangle.

Superolaterally Epigastric vessels Medially Lateral edge of rectus muscle

Inferiorly Inguinal ligament

The boundaries of Hesselbachs triangle are commonly tested and illustrated below

Image sourced from Wikipedia

Next question

Question 516 of 560

Which of the following muscles is not innervated by the ansa cervicalis?

Sternohyoid

Mylohyoid

Omohyoid

Sternothyroid

None of the above

Ansa cervicalis muscles:

GHost THought SOmeone Stupid Shot Irene

GenioHyoid ThyroidHyoid Superior Omohyoid SternoThyroid SternoHyoid Inferior Omohyoid

Mylohyoid is innervated by the mylohyoid branch of the inferior alveolar nerve.

Please rate this question:

Discuss and give feedback

Next question

Ansa cervicalis Superior Branch of C1 anterolateral to carotid sheath root

Inferior root Derived from C2 and C3 roots, passes posterolateral to the internal jugular vein (may lie either deep or superficial to it)

Innervation Sternohyoid Sternothyroid Omohyoid

The ansa cervicalis lies anterior to the carotid sheath. The nerve supply to the inferior strap muscles enters at their inferior aspect. Therefore when dividing these muscles to expose a large goitre, the muscles should be divided in their upper half.

Image sourced from Wikipedia

Question 517 of 560

A 58 year old lady presents with a mass in the upper outer quadrant of the right breast. Which of the following statements relating to the breast is untrue?

The internal mammary artery provides the majority of its arterial supply

Nipple retraction may occur as a result of tumour infiltration of the clavipectoral fascia

The internal mammary artery is a branch of the subclavian artery

Up to 70% of lymphatic drainage is to the ipsilateral axillary nodes

None of the above

Both skin dimpling and nipple retraction are features of breast malignancy. However, they usually occur as a result of tumour infiltration of the breast and ducts respectively. The clavipectoral fascia encases the axillary contents. The lymphatic drainage of the breast is to the axilla and also to the internal mammary chain. The breast is well vascularised and the internal mammary artery is a branch of the subclavian artery. Please rate this question:

Discuss and give feedback Next question Breast

The breast itself lies on a layer of pectoral fascia and the following muscles: 1. Pectoralis major 2. Serratus anterior 3. External oblique

Image showing the topography of the female breast

Image sourced from Wikipedia

Breast anatomy Nerve supply Branches of intercostal nerves from T4-T6.

Arterial supply  Internal mammary (thoracic) artery  External mammary artery (laterally)  Anterior intercostal arteries  Thoraco-acromial artery

Venous drainage Superficial venous plexus to subclavian, axillary and intercostal veins.

Lymphatic  70% Axillary nodes drainage  Internal mammary chain  Other lymphatic sites such as deep cervical and supraclavicular fossa (later in disease)

Next question

Question 518 of 560

Where are accessory spleens not found?

Gonads

Tail of pancreas

Greater omentum

Splenorenal ligament

Ureter

Accessory spleens

- 10% population - 1 cm size - locations: hilum of the spleen, tail of the pancreas, along the splenic vessels, in the gastrosplenic ligament, the splenorenal ligament, the walls of the stomach or intestines, the , the , the gonads Please rate this question:

Discuss and give feedback Next question Spleen

The spleen is located in the left upper quadrant of the abdomen and its size can vary depending upon the amount of blood it contains. The typical adult spleen is 12.5cm long and 7.5cm wide. The usual weight of the adult spleen is 150g. The exact position of the spleen can vary with respiratory activity, posture and the state of surrounding viscera. It usually lies obliquely with its long axis aligned to the 9th, 10th and 11th ribs. It is separated from these ribs by both diaphragm and pleural cavity. The normal spleen is not palpable.

The shape of the spleen is influenced by the state of the colon and stomach. Gastric distension will cause the spleen to resemble the shape of an orange segment. Colonic distension will cause it to become more tetrahedral.

The spleen is almost entirely covered by peritoneum, which adheres firmly to its capsule. Recesses of the greater sac separate it from the stomach and kidney. It develops from the upper dorsal mesogastrium, remaining connected to the posterior abdominal wall and stomach by two folds of peritoneum; the lienorenal ligament and gastrosplenic ligament. The lienorenal ligament is derived from peritoneum where the wall of the general peritoneum meets the omental bursa between the left kidney and spleen; the splenic vessels lie in its layers. The gastrosplenic ligament also has two layers, formed by the meeting of the walls of the greater sac and omental bursa between spleen and stomach, the short gastric and left gastroepiploic branches of the splenic artery pass in its layers. Laterally, the spleen is in contact with the phrenicocolic ligament.

Relations

Superiorly Diaphragm

Anteriorly Gastric impression

Posteriorly Kidney

Inferiorly Colon

Tail of pancreas and splenic vessels (splenic artery divides here, branches pass to the Hilum white pulp transporting plasma)

Contents

White Immune function. Contains central trabecular artery. The germinal centres are supplied pulp by arterioles called penicilliary radicles.

Red pulp Filters abnormal red blood cells.

Function

 Filtration of abnormal blood cells and foreign bodies such as bacteria.  Immunity: IgM. Production of properdin, and tuftsin which help target fungi and bacteria for phagocytosis.  Haematopoiesis: up to 5th month gestation or in haematological disorders.  Pooling: storage of 40% platelets.  Iron reutilisation  Storage monocytes

Disorders of the spleen Massive splenomegaly

 Myelofibrosis  Chronic myeloid leukaemia  Visceral leishmaniasis (kala-azar)  Malaria  Gaucher's syndrome

Other causes (as above plus)

 Portal hypertension e.g. secondary to cirrhosis  Lymphoproliferative disease e.g. CLL, Hodgkin's  Haemolytic anaemia  Infection: hepatitis, glandular fever  Infective endocarditis  Sickle-cell*, thalassaemia  Rheumatoid arthritis (Felty's syndrome)

*the majority of adult patients with sickle-cell will have an atrophied spleen due to repeated infarction Next question

Question 519-521 of 560

Theme: Nerve injury

A. Median nerve B. Ulnar nerve C. Radial nerve D. Anterior interosseous nerve E. Posterior interosseous nerve F. Axillary nerve G. Musculocutaneous nerve

Please select the nerve at risk of injury in each scenario. Each option may be used once, more than once or not at all.

519. A 43 year old typist presents with pain at the dorsal aspect of the upper part of her forearm. She also complains of weakness when extending her fingers. On examination triceps and supinator are both functioning normally. There is weakness of most of the extensor muscles. However, there is no sensory deficit.

You answered Median nerve

The correct answer is Posterior interosseous nerve

The radial nerve may become entrapped in the "arcade of Frohse" which is a superficial part of the supinator muscle which overlies the posterior interosseous nerve. This nerve is entirely muscular and articular in its distribution. It passes postero-inferiorly and gives branches to extensor carpi radialis brevis and supinator. It enters supinator and curves around the lateral and posterior surfaces of the radius. On emerging from the supinator the posterior interosseous nerve lies between the superficial extensor muscles and the lowermost fibres of supinator. It then gives branches to the extensors.

520. A 28 year teacher reports difficulty with writing. There is no sensory loss. She is known to have an aberrant Gantzer muscle.

You answered Median nerve

The correct answer is Anterior interosseous nerve

Anterior interosseous lesions occur due to fracture, or rarely due to compression. The Gantzer muscle is an aberrant accessory of the flexor pollicis longus and is a risk factor for anterior interosseous nerve compression. Remember loss of pincer grip and normal sensation indicates an interosseous nerve lesion. 521. A 35 year tennis player attends reporting tingling down his arm. He says that his 'funny bone' was hit very hard by a tennis ball. There is weakness of abduction and adduction of his extended fingers.

You answered Median nerve

The correct answer is Ulnar nerve

Theme from September 2012 exam The ulnar nerve arises from the medial cord of the brachial plexus (C8, T1 and contribution from C7). The nerve descends between the axillary artery and vein, posterior to the cutaneous nerve of the forearm and then lies anterior to triceps on the medial side of the brachial artery. In the distal half of the arm it passes through the medial intermuscular septum, and continues between this structure and the medial head of triceps to enter the forearm between the medial epicondyle of the humerus and the olecranon. It may be injured at this site in this scenario.

Please rate this question:

Discuss and give feedback Next question Brachial plexus

Origin Anterior rami of C5 to T1

Sections of the  Roots, trunks, divisions, cords, branches plexus  Mnemonic:Real Teenagers Drink Cold Beer

Roots  Located in the posterior triangle  Pass between scalenus anterior and medius

Trunks  Located posterior to middle third of clavicle  Upper and middle trunks related superiorly to the subclavian artery  Lower trunk passes over 1st rib posterior to the subclavian artery

Divisions Apex of axilla

Cords Related to axillary artery

Diagram illustrating the branches of the brachial plexus

Image sourced from Wikipedia

Cutaneous sensation of the upper limb

Image sourced from Wikipedia

Next question

Question 522 of 560

A 72 year old man is undergoing a left pneumonectomy for carcinoma of the bronchus. As the surgeons approach the root of the lung, which structure will lie most anteriorly (in the anatomical plane)?

Vagus nerve

Phrenic nerve

Bronchus

Pulmonary vein

Pulmonary artery

The phrenic nerve is the most anteriorly located structure in the lung root. The vagus nerve lies most posteriorly. Please rate this question:

Discuss and give feedback Next question Lung anatomy

The right lung is composed of 3 lobes divided by the oblique and transverse fissures. The left lung has two lobes divided by the oblique fissure.The apex of both lungs is approximately 4cm superior to the sterno-costal joint of the first rib. Immediately below this is a sulcus created by the subclavian artery.

Peripheral contact points of the lung

 Base: diaphragm  Costal surface: corresponds to the cavity of the chest  Mediastinal surface: Contacts the mediastinal pleura. Has the cardiac impression. Above and behind this concavity is a triangular depression named the hilum, where the structures which form the root of the lung enter and leave the viscus. These structures are invested by pleura, which, below the hilum and behind the pericardial impression, forms the pulmonary ligament

Right lung Above the hilum is the azygos vein; Superior to this is the groove for the superior vena cava and right innominate vein; behind this, and nearer the apex, is a furrow for the innominate artery. Behind the hilum and the attachment of the pulmonary ligament is a vertical groove for the oesophagus; In front and to the right of the lower part of the oesophageal groove is a deep concavity for the extrapericardiac portion of the inferior vena cava.

The root of the right lung lies behind the superior vena cava and the right atrium, and below the azygos vein.

The right main bronchus is shorter, wider and more vertical than the left main bronchus and therefore the route taken by most foreign bodies.

Image sourced from Wikipedia

Left lung Above the hilum is the furrow produced by the aortic arch, and then superiorly the groove accommodating the left subclavian artery; Behind the hilum and pulmonary ligament is a vertical groove produced by the descending aorta, and in front of this, near the base of the lung, is the lower part of the oesophagus.

The root of the left lung passes under the aortic arch and in front of the descending aorta.

Image sourced from Wikipedia

Inferior borders of both lungs

 6th rib in mid clavicular line  8th rib in mid axillary line  10th rib posteriorly

The pleura runs two ribs lower than the corresponding lung level.

Bronchopulmonary segments Segment number Right lung Left lung

1 Apical Apical

2 Posterior Posterior

3 Anterior Anterior

4 Lateral Superior lingular

5 Medial Inferior lingular

6 Superior (apical) Superior (apical) Segment number Right lung Left lung

7 Medial basal Medial basal

8 Anterior basal Anterior basal

9 Lateral basal Lateral basal

10 Posterior basal Posterior basal

Next question

Question 523 of 560

A 56 year old man is undergoing an anterior resection for a carcinoma of the rectum. Which of the structures below is least likely to be encountered during the mobilisation of the anterior rectum?

Denonvilliers' fascia

Middle sacral artery

Bladder

Rectovesical pouch

Seminal vesicles

With the exception of the middle sacral artery all of the other structures lie anterior to the rectum. They may all be palpated during digital rectal examination. Please rate this question:

Discuss and give feedback Next question Rectum

The rectum is approximately 12 cm long. It is a capacitance organ. It has both intra and extraperitoneal components. The transition between the sigmoid colon is marked by the disappearance of the tenia coli.The extra peritoneal rectum is surrounded by mesorectal fat that also contains lymph nodes. This mesorectal fatty layer is removed surgically during rectal cancer surgery (Total Mesorectal Excision). The fascial layers that surround the rectum are important clinical landmarks, anteriorly lies the fascia of Denonvilliers. Posteriorly lies Waldeyers fascia.

Extra peritoneal rectum

 Posterior upper third  Posterior and lateral middle third  Whole lower third

Relations Anteriorly (Males) Rectovesical pouch Bladder Prostate Seminal vesicles

Anteriorly (Females) Recto-uterine pouch (Douglas) Cervix Vaginal wall

Posteriorly Sacrum Coccyx Middle sacral artery

Laterally Levator ani Coccygeus

Arterial supply Superior rectal artery

Venous drainage Superior rectal vein

Lymphatic drainage

 Mesorectal lymph nodes (superior to dentate line)  Internal iliac and then para-aortic nodes  Inguinal nodes (inferior to dentate line)

Next question

Question 524 of 560

In relation to the middle cranial fossa, which of the following statements relating to the foramina is incorrect?

The foramen rotundum transmits the maxillary nerve

The foramen lacerum is closely related to the internal carotid artery

The foramen spinosum lies posterolateral to the foramen ovale

The foramen ovale transmits the middle meningeal artery

The foramen rotundum lies anteromedial to the foramen ovale

Theme addressed in 2010 and 2011 exam The foramen spinosum transmits the middle meningeal artery. The foramen ovale transmits the mandibular nerve. As the foramina weaken the bone, a fracture at this site is not uncommon.

Please rate this question:

Discuss and give feedback

Next question

Foramina of the base of the skull

Foramen Location Contents

Foramen ovale Sphenoid Otic ganglion V3 (Mandibular nerve:3rd branch of Foramen Location Contents

bone trigeminal) Accessory meningeal artery Lesser petrosal nerve Emissary veins

Foramen spinosum Sphenoid Middle meningeal artery bone Meningeal branch of the Mandibular nerve

Foramen rotundum Sphenoid Maxillary nerve (V2) bone

Foramen lacerum/ Sphenoid Base of the medial pterygoid plate. carotid canal bone Internal carotid artery* Nerve and artery of the pterygoid canal

Jugular foramen Temporal Anterior: inferior petrosal sinus bone Intermediate: glossopharyngeal, vagus, and accessory nerves. Posterior: sigmoid sinus (becoming the internal jugular vein) and some meningeal branches from the occipital and ascending pharyngeal arteries.

Foramen magnum Occipital Anterior and posterior spinal arteries bone Vertebral arteries Medulla oblongata

Stylomastoid Temporal Stylomastoid artery foramen bone Facial nerve

Superior orbital Sphenoid Oculomotor nerve (III) fissure bone Recurrent meningeal artery Trochlear nerve (IV) Lacrimal, frontal and nasociliary branches of ophthalmic nerve (V1) Foramen Location Contents

Abducent nerve (VI) Superior ophthalmic vein

*= In life the foramen lacerum is occluded by a cartilagenous plug. The ICA initially passes into the carotid canal which ascends superomedially to enter the cranial cavity through the foramen lacerum.

Base of skull anatomical overview

Image sourced from Wikipedia

Next question

Question 525 of 560

During an operation for varicose veins the surgeons are mobilising the long saphenous vein. Near its point of entry to the femoral vein an artery is injured and bleeding is encountered. From where is the bleeding most likely to originate?

Femoral artery

Profunda femoris artery

Superficial circumflex iliac artery

Superficial epigastric artery

Deep external pudendal artery

Theme from 2011 Exam The deep external pudendal artery is a branch of the SFA and it runs medially under the long saphenous vein near its point of union with the femoral vein. The superficial external pudendal artery lies superior to the SFJ. Neither vessel is functionally important and if injured they are best ligated. Please rate this question:

Discuss and give feedback Next question Femoral triangle anatomy

Boundaries Superiorly Inguinal ligament

Laterally Sartorius

Medially Adductor longus

Floor Iliopsoas, adductor longus and pectineus Roof  Fascia lata and Superficial fascia  Superficial inguinal lymph nodes (palpable below the inguinal ligament)  Long saphenous vein

Image sourced from Wikipedia

Contents

 Femoral vein (medial to lateral)  Femoral artery-pulse palpated at the mid inguinal point  Femoral nerve  Deep and superficial inguinal lymph nodes  Lateral cutaneous nerve  Great saphenous vein  Femoral branch of the genitofemoral nerve

Next question

Question 526 of 560

A 78 year old man is lifting a heavy object when a feels a pain in his forearm and is unable to continue. He has a swelling over his upper forearm. An MRI scan shows a small cuff of tendon still attached to the radial tuberosity consistent with a recent tear. Which of the following muscles has been injured?

Pronator teres

Supinator

Aconeus

Brachioradialis

Biceps brachii

Biceps inserts into the radial tuberosity. Distal injuries of this muscle are rare but are reported and are clinically more important than more proximal ruptures. Please rate this question:

Discuss and give feedback Next question Radius

The radius is one of the two long forearm bones that extends from the lateral side of the elbow to the thumb side of the wrist. It has two expanded ends, of which the distal end is the larger. Key points relating to its topography and relations are outlined below;

Upper end

 Articular cartilage- covers medial > lateral side  Articulates with radial notch of the ulna by the annular ligament  Muscle attachment- biceps brachii at the tuberosity

Shaft Muscle attachment Upper third of the body Supinator Flexor digitorum superficialis Flexor pollicis longus

Middle third of the body Pronator teres

Lower quarter of the body Pronator quadratus Tendon of supinator longus

Lower end

 Quadrilateral  Anterior surface- capsule of wrist joint  Medial surface- head of ulna  Lateral surface- ends in the styloid process  Posterior surface: 3 grooves containing:

1. Tendons of extensor carpi radialis longus and brevis 2. Tendon of extensor pollicis longus 3. Tendon of extensor indicis

Image sourced from Wikipedia

Next question

Question 527 of 560

What is embryological origin of the pulmonary artery?

First pharyngeal arch

Second pharyngeal arch

Fourth pharyngeal arch

Fifth pharyngeal arch

Sixth pharyngeal arch

Theme from September 2011 Exam Theme from September 2012 Exam The proximal part of the sixth right pharyngeal arch gives origin to the right pulmonary artery. The distal part gives origin to the left pulmonary artery and the ductus arteriosus.

Please rate this question:

Discuss and give feedback

Next question

Pharyngeal arches

These develop during the fourth week of embryonic growth from a series of mesodermal outpouchings of the developing pharynx. They develop and fuse in the ventral midline. Pharyngeal pouches form on the endodermal side between the arches. There are 6 pharyngeal arches, the fifth does not contribute any useful structures and often fuses with the sixth arch.

Pharyngeal arches

Pharyngeal Muscular Skeletal arch contributions contributions Endocrine Artery Nerve

First Muscles of Maxilla n/a Maxillary Mandibular mastication Meckels External Anterior belly of cartilage carotid digastric Incus Mylohyoid Malleus Tensor tympanic Tensor veli palatini

Second Buccinator Stapes n/a Inferior Facial Platysma Styloid process branch of Muscles of facial Lesser horn superior expression and upper thyroid artery Stylohyoid body of hyoid Stapedial Posterior belly of artery digastric Stapedius

Third Stylopharyngeus Greater horn Thymus Common and Glossopharyngeal and lower part Inferior internal of hyoid parathyroids carotid

Fourth Cricothyroid Thyroid and Superior Right- Vagus All intrinsic epiglottic parathyroids subclavian muscles of the soft cartilages artery, Left- palate aortic arch

Sixth All intrinsic Cricoid, n/a Right - Vagus and muscles of the arytenoid and Pulmonary recurrent larynx (except corniculate artery, Left- laryngeal nerve cricothyroid) cartilages Pulmonary artery and Pharyngeal Muscular Skeletal arch contributions contributions Endocrine Artery Nerve

ductus arteriosus

Next question

Question 528 of 560

A 53 year old lady presents with pain and discomfort in her hand. She works as a typist and notices that the pain is worst when she is working. She also suffers symptoms at night. Her little finger is less affected by the pain. Which of the nerves listed below is most likely to be affected?

Radial

Median

Ulnar

Anterior interosseous nerve

Posterior interosseous nerve

Motor supply: LOAF

L ateral 2 lumbricals O pponens pollicis A bductor pollicisbrevis F lexor pollicis brevis

Theme from September 2015 Exam The most likely diagnosis here is carpal tunnel syndrome, the median nerve is compressed in the wrist and symptoms usually affect the fingers and wrist either at night or when the hand is being used (e.g. as a typist). Please rate this question:

Discuss and give feedback Next question Median nerve

The median nerve is formed by the union of a lateral and medial root respectively from the lateral (C5,6,7) and medial (C8 and T1) cords of the brachial plexus; the medial root passes anterior to the third part of the axillary artery. The nerve descends lateral to the brachial artery, crosses to its medial side (usually passing anterior to the artery). It passes deep to the bicipital aponeurosis and the median cubital vein at the elbow. It passes between the two heads of the pronator teres muscle, and runs on the deep surface of flexor digitorum superficialis (within its fascial sheath). Near the wrist it becomes superficial between the tendons of flexor digitorum superficialis and flexor carpi radialis, deep to palmaris longus tendon. It passes deep to the flexor retinaculum to enter the palm, but lies anterior to the long flexor tendons within the carpal tunnel.

Branches Region Branch

Upper arm No branches, although the nerve commonly communicates with the musculocutaneous nerve

Forearm Pronator teres Flexor carpi radialis Palmaris longus Flexor digitorum superficialis Flexor pollicis longus Flexor digitorum profundus (only the radial half)

Distal Palmar cutaneous branch forearm

Hand Motor supply (LOAF) (Motor)

 Lateral 2 lumbricals  Opponens pollicis  Abductor pollicis brevis  Flexor pollicis brevis

Hand  Over thumb and lateral 2 ½ fingers (Sensory)  On the palmar aspect this projects proximally, on the dorsal aspect only the distal regions are innervated with the radial nerve providing the more proximal cutaneous innervation.

Patterns of damage Damage at wrist

 e.g. carpal tunnel syndrome  paralysis and wasting of thenar eminence muscles and opponens pollicis (ape hand deformity)  sensory loss to palmar aspect of lateral (radial) 2 ½ fingers

Damage at elbow, as above plus:

 unable to pronate forearm  weak wrist flexion  ulnar deviation of wrist

Anterior interosseous nerve (branch of median nerve)

 leaves just below the elbow  results in loss of pronation of forearm and weakness of long flexors of thumb and index finger

Topography of the median nerve

Image sourced from Wikipedia

Next question Question 529 of 560

Which of the following muscles lies medial to the long thoracic nerve?

Serratus anterior

Latissimus dorsi

Pectoralis major

Pectoralis minor

None of the above

Theme from 2009 Exam Please rate this question:

Discuss and give feedback Next question Long thoracic nerve

 Derived from ventral rami of C5, C6, and C7 (close to their emergence from intervertebral foramina)  It runs downward and passes either anterior or posterior to the middle scalene muscle  It reaches upper tip of serratus anterior muscle and descends on outer surface of this muscle, giving branches into it  Winging of Scapula occurs in long thoracic nerve injury (most common) or from spinal accessory nerve injury (which denervates the trapezius) or a dorsal scapular nerve injury

Next question

Question 530 of 560

The thebesian veins contribute to the venous drainage of the heart. Into which of the following structures do they primarily drain?

Great cardiac vein

Atrium

Superior vena cava

Oblique vein

Small cardiac vein

The thebesian veins are numerous small veins running over the surface of the heart they drain into the heart itself. Usually this is to the atrium directly. Please rate this question:

Discuss and give feedback Next question Heart anatomy

The walls of each cardiac chamber comprise:

 Epicardium  Myocardium  Endocardium

Cardiac muscle is attached to the cardiac fibrous skeleton.

Relations The heart and roots of the great vessels within the pericardial sac are related anteriorly to the sternum, medial ends of the 3rd to 5th ribs on the left and their associated costal cartilages. The heart and pericardial sac are situated obliquely two thirds to the left and one third to the right of the median plane.

The pulmonary valve lies at the level of the left third costal cartilage. The mitral valve lies at the level of the fourth costal cartilage.

Coronary sinus This lies in the posterior part of the coronary groove and receives blood from the cardiac veins. The great cardiac vein lies at its left and the middle and small cardiac veins lie on its right. The smallest cardiac vein (anterior cardiac vein) drains into the right atrium directly.

Aortic sinus Right coronary artery arises from the right aortic sinus, the left is derived from the left aortic sinus, which lies posteriorly.

Right and left ventricles

Structure Left Ventricle

A-V Valve Mitral (double leaflet)

Walls Twice as thick as right

Trabeculae carnae Much thicker and more numerous

Right coronary artery The RCA supplies:

 Right atrium  Diaphragmatic part of the left ventricle  Usually the posterior third of the interventricular septum  The sino atrial node (60% cases)  The atrio ventricular node (80% cases)

Left coronary artery The LCA supplies:

 Left atrium  Most of left ventricle  Part of the right ventricle  Anterior two thirds of the inter ventricular septum  The sino atrial node (remaining 40% cases)

Innervation of the heart Autonomic nerve fibres from the superficial and deep cardiac plexus. These lie anterior to the bifurcation of the trachea, posterior to the ascending aorta and superior to the bifurcation of the pulmonary trunk. The parasympathetic supply to the heart is from presynaptic fibres of the vagus nerves.

Valves of the heart Mitral valve Aortic valve Pulmonary valve Tricuspid valve

2 cusps 3 cusps 3 cusps 3 cusps

First heart sound Second heart Second heart First heart sound sound sound

1 anterior cusp 2 anterior cusps 2 anterior cusps 2 anterior cusps

Attached to chordae No chordae No chordae Attached to chordae tendinae tendinae

Next question

Question 531 of 560

Which of the following is not contained within the deep posterior compartment of the lower leg?

Tibialis posterior muscle

Posterior tibial artery

Tibial nerve

Sural nerve

Flexor hallucis longus

The deep posterior compartment lies anterior to soleus. The sural nerve is superficially sited and therefore not contained within it.

Please rate this question:

Discuss and give feedback

Next question

Lower limb- Muscular compartments

Anterior compartment

Muscle Nerve Action

Tibialis anterior Deep peroneal nerve Dorsiflexes ankle joint, inverts foot Muscle Nerve Action

Extensor digitorum longus Deep peroneal nerve Extends lateral four toes, dorsiflexes ankle joint

Peroneus tertius Deep peroneal nerve Dorsiflexes ankle, everts foot

Extensor hallucis longus Deep peroneal nerve Dorsiflexes ankle joint, extends big toe

Peroneal compartment

Muscle Nerve Action

Peroneus longus Superficial peroneal nerve Everts foot, assists in plantar flexion

Peroneus brevis Superficial peroneal nerve Plantar flexes the ankle joint

Superficial posterior compartment

Nerve Action

Gastrocnemius Tibial Plantar flexes the foot, may also flex nerve the knee

Soleus Tibial Plantar flexor nerve

Deep posterior compartment

Muscle Nerve Action Muscle Nerve Action

Flexor digitorum longus Tibial Flexes the lateral four toes

Flexor hallucis longus Tibial Flexes the great toe

Tibialis posterior Tibial Plantar flexor, inverts the foot

Next question

Question 532 of 560

When performing minor surgery in the scalp, which of the following regions is considered a danger area as regards spread of infection into the CNS?

Aponeurosis epicranialis

Skin

Pericranium

Connective tissue

Loose areolar tissue

This area is most dangerous as infections can spread easily. The emissary veins that drain this area may allow sepsis to spread to the cranial cavity. Please rate this question:

Discuss and give feedback Next question Head injury

Patients who suffer head injuries should be managed according to ATLS principles and extra cranial injuries should be managed alongside cranial trauma. Inadequate cardiac output will compromise CNS perfusion irrespective of the nature of the cranial injury.

Types of traumatic brain injury

Bleeding into the space between the dura mater and the skull. Often results from acceleration-deceleration trauma or a blow to the side of the head. The majority of extradural haematomas occur in the temporal region where skull fractures cause a rupture of the middle meningeal artery.

Extradural Features haematoma

 Raised intracranial pressure  Some patients may exhibit a lucid interval Bleeding into the outermost meningeal layer. Most commonly occur around the frontal and parietal lobes. May be either acute or chronic. Subdural haematoma Risk factors include old age and alcoholism.

Slower onset of symptoms than a extradural haematoma.

Usually occurs spontaneously in the context of a ruptured cerebral aneurysm, but Subarachnoid may be seen in association with other injuries when a patient has sustained a haemorrhage traumatic brain injury.

Pathophysiology

 Primary brain injury may be focal (contusion/ haematoma) or diffuse (diffuse axonal injury)  Diffuse axonal injury occurs as a result of mechanical shearing following deceleration, causing disruption and tearing of axons  Intra-cranial haematomas can be extradural, subdural or intracerebral, while contusions may occur adjacent to (coup) or contralateral (contre-coup) to the side of impact  Secondary brain injury occurs when cerebral oedema, ischaemia, infection, tonsillar or tentorial herniation exacerbates the original injury. The normal cerebral auto regulatory processes are disrupted following trauma rendering the brain more susceptible to blood flow changes and hypoxia  The Cushings reflex (hypertension and bradycardia) often occurs late and is usually a pre terminal event

Management

 Where there is life threatening rising ICP such as in extra dural haematoma and whilst theatre is prepared or transfer arranged use of IV mannitol/ frusemide may be required.  Diffuse cerebral oedema may require decompressive craniotomy  Exploratory Burr Holes have little management in modern practice except where scanning may be unavailable and to thus facilitate creation of formal craniotomy flap  Depressed skull fractures that are open require formal surgical reduction and debridement, closed injuries may be managed non operatively if there is minimal displacement.  ICP monitoring is appropriate in those who have GCS 3-8 and normal CT scan.  ICP monitoring is mandatory in those who have GCS 3-8 and abnormal CT scan.  Hyponatraemia is most likely to be due to syndrome of inappropriate ADH secretion.  Minimum of cerebral perfusion pressure of 70mmHg in adults.  Minimum cerebral perfusion pressure of between 40 and 70 mmHg in children.

Interpretation of pupillary findings in head injuries Pupil size Light response Interpretation Pupil size Light response Interpretation

Unilaterally dilated Sluggish or fixed 3rd nerve compression secondary to tentorial herniation

Bilaterally dilated Sluggish or fixed  Poor CNS perfusion  Bilateral 3rd nerve palsy

Unilaterally dilated or Cross reactive (Marcus - Optic nerve injury equal Gunn)

Bilaterally constricted May be difficult to  Opiates assess  Pontine lesions  Metabolic encephalopathy

Unilaterally Preserved Sympathetic pathway disruption constricted Next question

Question 533 of 560

Which of the following structures are at risk of direct injury following a fracture dislocation of the femoral condyles?

Popliteal artery

Sciatic nerve

Plantaris muscle

Tibial artery

Tibial nerve

The heads of gastrocnemius will contract to pull the fracture segment posteriorly. The popliteal artery lies against the bone and may be damaged or compressed. Please rate this question:

Discuss and give feedback Next question Popliteal fossa

Boundaries of the popliteal fossa Laterally Biceps femoris above, lateral head of gastrocnemius and plantaris below

Medially Semimembranosus and semitendinosus above, medial head of gastrocnemius below

Floor Popliteal surface of the femur, posterior ligament of knee joint and popliteus muscle

Roof Superficial and deep fascia

Image showing the popliteal fossa

© Image provided by the University of Sheffield

Contents

 Popliteal artery and vein  Small saphenous vein  Common peroneal nerve  Tibial nerve  Posterior cutaneous nerve of the thigh  Genicular branch of the obturator nerve  Lymph nodes

Next question

Question 534 of 560

A 25 year old man is being catheterised, prior to a surgical procedure. As the catheter enters the prostatic urethra which of the following changes will occur?

Resistance will increase significantly

Resistance will increase slightly

It will lie horizontally

Resistance will decrease

It will deviate laterally

Theme from September 2011 Exam The prostatic urethra is much wider than the membranous urethra and therefore resistance will decrease. The prostatic urethra is inclined vertically. Please rate this question:

Discuss and give feedback Next question Prostate gland

The prostate gland is approximately the shape and size of a walnut and is located inferior to the bladder. It is separated from the rectum by Denonvilliers fascia and its blood supply is derived from the internal iliac vessels (via inferior vesical artery). The internal sphincter lies at the apex of the gland and may be damaged during prostatic surgery, affected individuals may complain of retrograde ejaculation.

Summary of prostate gland Arterial supply Inferior vesical artery (from internal iliac)

Venous drainage Prostatic venous plexus (to paravertebral veins) Lymphatic Internal iliac nodes drainage

Innervation Inferior hypogastric plexus

Dimensions  Transverse diameter (4cm)  AP diameter (2cm)  Height (3cm)

Lobes  Posterior lobe: posterior to urethra  Median lobe: posterior to urethra, in between ejaculatory ducts  Lateral lobes x 2  Isthmus

Zones  Peripheral zone: subcapsular portion of posterior prostate. Most prostate cancers are here  Central zone  Transition zone  Stroma

Relations Pubic symphysis Anterior Prostatic venous plexus

Posterior Denonvilliers fascia Rectum Ejaculatory ducts

Lateral Venous plexus (lies on prostate) Levator ani (immediately below the puboprostatic ligaments)

Image sourced from Wikipedia

Next question

Question 535 of 560

A 24 year female is admitted to A&E with tingling of her hand after a fall. She is found to have a fracture of the medial epicondyle. What is the most likely nerve lesion?

Ulnar nerve

Radial nerve

Median nerve

Axillary nerve

Cutaneous nerve

The radial nerve is located near the lateral epicondyle. Please rate this question:

Discuss and give feedback Next question Ulnar nerve

Origin

 C8, T1

Supplies (no muscles in the upper arm)

 Flexor carpi ulnaris  Flexor digitorum profundus  Flexor digiti minimi  Abductor digiti minimi  Opponens digiti minimi  Adductor pollicis  Interossei muscle  Third and fourth lumbricals  Palmaris brevis

Path

 Posteromedial aspect of upper arm to flexor compartment of forearm, then along the ulnar. Passes beneath the flexor carpi ulnaris muscle, then superficially through the flexor retinaculum into the palm of the hand.

Image sourced from Wikipedia

Branches Branch Supplies

Muscular branch Flexor carpi ulnaris Medial half of the flexor digitorum profundus Branch Supplies

Palmar cutaneous branch (Arises near the Skin on the medial part of the palm middle of the forearm)

Dorsal cutaneous branch Dorsal surface of the medial part of the hand

Superficial branch Cutaneous fibres to the anterior surfaces of the medial one and one-half digits

Deep branch Hypothenar muscles All the interosseous muscles Third and fourth lumbricals Adductor pollicis Medial head of the flexor pollicis brevis

Effects of injury Damage at the wrist  Wasting and paralysis of intrinsic hand muscles (claw hand)  Wasting and paralysis of hypothenar muscles  Loss of sensation medial 1 and half fingers

Damage at the elbow  Radial deviation of the wrist  Clawing less in 4th and 5th digits

Next question

Question 536 of 560

During a gangland gunfight a man is shot in the chest. The bullet passes through the posterior mediastinum (from left to right). Which of the following structures is least likely to be injured

Thoracic duct

Oesophagus

Vagus nerve

Descending thoracic aorta

Arch of the azygos vein

The arch of the azygos vein lies in the middle mediastinum. Please rate this question:

Discuss and give feedback Next question Mediastinum

Region between the pulmonary cavities. It is covered by the mediastinal pleura. It does not contain the lungs. It extends from the thoracic inlet superiorly to the diaphragm inferiorly.

Mediastinal regions

 Superior mediastinum (between manubriosternal angle and T4/5)  Middle mediastinum  Posterior mediastinum  Anterior mediastinum

Region Contents Region Contents

Superior mediastinum  Superior vena cava  Brachiocephalic veins  Arch of aorta  Thoracic duct  Trachea  Oesophagus  Thymus  Vagus nerve  Left recurrent laryngeal nerve  Phrenic nerve

Anterior mediastinum  Thymic remnants  Lymph nodes  Fat

Middle mediastinum  Pericardium  Heart  Aortic root  Arch of azygos vein  Main bronchi

Posterior mediastinum  Oesophagus  Thoracic aorta  Azygos vein  Thoracic duct  Vagus nerve  Sympathetic nerve trunks  Splanchnic nerves

Next question

Question 537 of 560

The space between the vocal cords is referred to as which of the following?

Piriform recess

Rima vestibuli

Vestibule

Glottis

Rima glottidis

The rima glottidis is the narrowest part of the laryngeal cavity. Please rate this question:

Discuss and give feedback Next question Larynx

The larynx lies in the anterior part of the neck at the levels of C3 to C6 vertebral bodies. The laryngeal skeleton consists of a number of cartilagenous segments. Three of these are paired; arytenoid, corniculate and cuneiform. Three are single; thyroid, cricoid and epiglottic. The cricoid cartilage forms a complete ring (the only one to do so). The laryngeal cavity extends from the laryngeal inlet to the level of the inferior border of the cricoid cartilage.

Divisions of the laryngeal cavity Laryngeal vestibule Superior to the vestibular folds

Laryngeal ventricle Lies between vestibular folds and superior to the vocal cords

Infraglottic cavity Extends from vocal cords to inferior border of the cricoid cartilage

The vocal folds (true vocal cords) control sound production. The apex of each fold projects medially into the laryngeal cavity. Each vocal fold includes:

 Vocal ligament  Vocalis muscle (most medial part of thyroarytenoid muscle)

The glottis is composed of the vocal folds, processes and rima glottidis. The rima glottidis is the narrowest potential site within the larynx, as the vocal cords may be completely opposed, forming a complete barrier.

Muscles of the larynx Muscle Origin Insertion Innervation Action

Posterior Posterior aspect Muscular process Recurrent Abducts vocal fold cricoarytenoid of lamina of of arytenoid Laryngeal cricoid

Lateral Arch of cricoid Muscular process Recurrent Adducts vocal fold cricoarytenoid of arytenoid laryngeal

Thyroarytenoid Posterior aspect Muscular process Recurrent Relaxes vocal fold of thyroid of arytenoid laryngeal cartilage

Transverse and Arytenoid Contralateral Recurrent Closure of oblique cartilage arytenoid laryngeal intercartilagenous arytenoids part of the rima glottidis

Vocalis Depression Vocal ligament Recurrent Relaxes posterior between lamina and vocal process laryngeal vocal ligament, tenses of thyroid of arytenoid anterior part cartilage cartilage

Cricothyroid Anterolateral Inferior margin External Tenses vocal fold part of cricoid and horn of laryngeal thyroid cartilage

Blood supply Arterial supply is via the laryngeal arteries, branches of the superior and inferior thyroid arteries. The superior laryngeal artery is closely related to the internal laryngeal nerve. The inferior laryngeal artery is related to the inferior laryngeal nerve. Venous drainage is via superior and inferior laryngeal veins, the former draining into the superior thyroid vein and the latter draining into the middle thyroid vein, or thyroid venous plexus.

Lymphatic drainage The vocal cords have no lymphatic drainage and this site acts as a lymphatic watershed. Supraglottic part Upper deep cervical nodes

Subglottic part Prelaryngeal and pretracheal nodes and inferior deep cervical nodes

The aryepiglottic fold and vestibular folds have a dense plexus of lymphatics associated with them and malignancies at these sites have a greater propensity for nodal metastasis.

Topography of the larynx

Image sourced from Wikipedia

Next question

Question 538 of 560

A 78 year old man develops a carcinoma of the scrotum. To which of the following lymph node groups may the tumour initially metastasise?

Para aortic

Obturator

Inguinal

Meso rectal

None of the above

The scrotum is drained by the inguinal nodes. Please rate this question:

Discuss and give feedback Next question Scrotal and testicular anatomy

Spermatic cord Formed by the vas deferens and is covered by the following structures: Layer Origin

Internal spermatic fascia Transversalis fascia

Cremasteric fascia From the fascial coverings of internal oblique

External spermatic fascia External oblique aponeurosis

Contents of the cord Vas deferens Transmits sperm and accessory gland secretions

Testicular artery Branch of abdominal aorta supplies testis and epididymis

Artery of vas deferens Arises from inferior vesical artery

Cremasteric artery Arises from inferior epigastric artery

Pampiniform plexus Venous plexus, drains into right or left testicular vein

Sympathetic nerve fibres Lie on arteries, the parasympathetic fibres lie on the vas

Genital branch of the genitofemoral Supplies cremaster nerve

Lymphatic vessels Drain to lumbar and para-aortic nodes

Scrotum

 Composed of skin and closely attached dartos fascia.  Arterial supply from the anterior and posterior scrotal arteries  Lymphatic drainage to the inguinal lymph nodes  Parietal layer of the tunica vaginalis is the innermost layer

Testes

 The testes are surrounded by the tunica vaginalis (closed peritoneal sac). The parietal layer of the tunica vaginalis adjacent to the internal spermatic fascia.  The testicular arteries arise from the aorta immediately inferiorly to the renal arteries.  The pampiniform plexus drains into the testicular veins, the left drains into the left renal vein and the right into the inferior vena cava.  Lymphatic drainage is to the para-aortic nodes.

Next question Question 539 of 560

A 63 year old man is undergoing an upper GI endoscopy for dysphagia. At 33 cm (from the incisors) a malignant looking stricture is encountered. The endoscopist attempts a balloon dilatation.Unfortunately the tumour splits through the oesophageal wall. Into which region will the oesophageal contents now drain?

Superior mediastinum

Posterior mediastinum

Middle mediastinum

Anterior mediastinum

Peritoneal cavity

At this position the oesophagus is still likely to be intrathoracic and located in the posterior mediastinum. Please rate this question:

Discuss and give feedback Next question Mediastinum

Region between the pulmonary cavities. It is covered by the mediastinal pleura. It does not contain the lungs. It extends from the thoracic inlet superiorly to the diaphragm inferiorly.

Mediastinal regions

 Superior mediastinum (between manubriosternal angle and T4/5)  Middle mediastinum  Posterior mediastinum  Anterior mediastinum

Region Contents

Superior mediastinum  Superior vena cava  Brachiocephalic veins  Arch of aorta  Thoracic duct  Trachea  Oesophagus  Thymus  Vagus nerve  Left recurrent laryngeal nerve  Phrenic nerve

Anterior mediastinum  Thymic remnants  Lymph nodes  Fat

Middle mediastinum  Pericardium  Heart  Aortic root  Arch of azygos vein  Main bronchi

Posterior mediastinum  Oesophagus  Thoracic aorta  Azygos vein  Thoracic duct  Vagus nerve  Sympathetic nerve trunks  Splanchnic nerves

Next question

Question 540 of 560

During a tricuspid valve repair the right atrium is opened, following establishment of cardiopulmonary bypass. Which of the following structures do not lie within the right atrium?

Crista terminalis

Tricuspid valve

Fossa ovalis

Trabeculae carnae

Musculi pectinati

Structures within the right atrium:

 Musculi pectinati  Crista terminalis  Opening of the coronary sinus  Fossa ovalis

The trabeculae carnae are located in the right ventricle. Please rate this question:

Discuss and give feedback Next question Heart anatomy

The walls of each cardiac chamber comprise:

 Epicardium  Myocardium  Endocardium

Cardiac muscle is attached to the cardiac fibrous skeleton.

Relations The heart and roots of the great vessels within the pericardial sac are related anteriorly to the sternum, medial ends of the 3rd to 5th ribs on the left and their associated costal cartilages. The heart and pericardial sac are situated obliquely two thirds to the left and one third to the right of the median plane.

The pulmonary valve lies at the level of the left third costal cartilage. The mitral valve lies at the level of the fourth costal cartilage.

Coronary sinus This lies in the posterior part of the coronary groove and receives blood from the cardiac veins. The great cardiac vein lies at its left and the middle and small cardiac veins lie on its right. The smallest cardiac vein (anterior cardiac vein) drains into the right atrium directly.

Aortic sinus Right coronary artery arises from the right aortic sinus, the left is derived from the left aortic sinus, which lies posteriorly.

Right and left ventricles

Structure Left Ventricle

A-V Valve Mitral (double leaflet)

Walls Twice as thick as right

Trabeculae carnae Much thicker and more numerous

Right coronary artery The RCA supplies:

 Right atrium  Diaphragmatic part of the left ventricle  Usually the posterior third of the interventricular septum  The sino atrial node (60% cases)  The atrio ventricular node (80% cases)

Left coronary artery The LCA supplies:

 Left atrium  Most of left ventricle  Part of the right ventricle  Anterior two thirds of the inter ventricular septum  The sino atrial node (remaining 40% cases)

Innervation of the heart Autonomic nerve fibres from the superficial and deep cardiac plexus. These lie anterior to the bifurcation of the trachea, posterior to the ascending aorta and superior to the bifurcation of the pulmonary trunk. The parasympathetic supply to the heart is from presynaptic fibres of the vagus nerves.

Valves of the heart Mitral valve Aortic valve Pulmonary valve Tricuspid valve

2 cusps 3 cusps 3 cusps 3 cusps

First heart sound Second heart Second heart First heart sound sound sound

1 anterior cusp 2 anterior cusps 2 anterior cusps 2 anterior cusps

Attached to chordae No chordae No chordae Attached to chordae tendinae tendinae

Next question

Question 541 of 560

Which of the following is a recognised tributary of the retromandibular vein?

Internal jugular vein

External jugular vein

Anterior temporal diploic vein

Maxillary vein

Inferior opthalmic vein

The retromandibular vein is formed from the union of the maxillary and superficial temporal veins. Please rate this question:

Discuss and give feedback Next question Retromandibular vein

 Formed by a union of the maxillary vein and superficial temporal vein  It descends through the parotid gland and bifurcates within it  The anterior division passes forwards to join the facial vein, the posterior division is one of the tributaries of the external jugular vein

Next question

Question 542 of 560

An 22 year old soldier is shot in the abdomen and amongst his various injuries is a major disruption to the abdominal aorta. There is torrential haemorrhage and the surgeons decide to control the aorta by placement of a vascular clamp immediately inferior to the diaphragm. Which of the following vessels may be injured in this maneouvre?

Inferior phrenic arteries

Superior phrenic arteries

Splenic artery

Renal arteries

Superior mesenteric artery

As the first branches of the abdominal aorta the inferior phrenic arteries are at greatest risk. The superior phrenic arteries lie in the thorax. The potential space at the level of the diaphragmatic hiatus is a potentially useful site for aortic occlusion. However, leaving the clamp applied for more than about 10 -15 minutes usually leads to poor outcomes.

Please rate this question:

Discuss and give feedback

Next question

Abdominal aorta

Abdominal aortic topography Origin T12

Termination L4

Posterior relations L1-L4 Vertebral bodies

Anterior relations Lesser omentum Left renal vein Inferior mesenteric vein Third part of duodenum Pancreas Parietal peritoneum

Right lateral relations Right crus of the diaphragm Cisterna chyli Azygos vein IVC (becomes posterior distally)

Left lateral relations 4th part of duodenum Duodenal-jejunal flexure Left sympathetic trunk

The abdominal aorta

Image sourced from Wikipedia

Next question

Question 543 of 560

Which of the following statements relating to the gallbladder is untrue?

The fundus is usually intra peritoneal

Arterial supply is from the cystic artery

The cystic artery is usually located in Calots triangle

Calots triangle may rarely contain an aberrant hepatic artery

Cholecystokinin causes relaxation of the gallbladder

CCK causes gallbladder contraction. Please rate this question:

Discuss and give feedback Next question Gallbladder

 Fibromuscular sac with capacity of 50ml  Columnar epithelium

Relations of the gallbladder Anterior Liver

Posterior  Covered by peritoneum  Transverse colon  1st part of the duodenum

Laterally Right lobe of liver

Medially Quadrate lobe of liver

Arterial supply Cystic artery (branch of Right hepatic artery)

Venous drainage Directly to the liver

Nerve supply Sympathetic- mid thoracic spinal cord, Parasympathetic- anterior vagal trunk

Common bile duct

Origin Confluence of cystic and common hepatic ducts

Relations at  Medially - Hepatic artery origin  Posteriorly- Portal vein

Relations distally  Duodenum - anteriorly  Pancreas - medially and laterally  Right renal vein - posteriorly

Arterial supply Branches of hepatic artery and retroduodenal branches of gastroduodenal artery

Hepatobiliary triangle

Medially Common hepatic duct

Inferiorly Cystic duct

Superiorly Inferior edge of liver

Contents Cystic artery

Relations of the gallbladder

© Image provided by the University of Sheffield

Next question

Question 544 of 560

Which of the following nerves is the primary source of innervation to the anterior scrotal skin?

Iliohypogastric nerve

Pudendal nerve

Ilioinguinal nerve

Femoral branch of the genitofemoral nerve

Obturator nerve

Theme from April 2012 Exam The pudendal nerve may innervate the posterior skin of the scrotum. The anterior innervation of the scrotum is primarily provided by the ilioinguinal nerve. The genital branch of the genitofemoral nerve provides a smaller contribution.

Please rate this question:

Discuss and give feedback

Next question

Scrotal sensation

The scrotum is innervated by the ilioinguinal nerve and the pudendal nerve. The ilioinguinal nerve arises from L1 and pierces the internal oblique muscle. It eventually passes through the superficial inguinal ring to innervate the anterior skin of the scrotum.

The pudendal nerve is the principal nerve of the perineum. It arises in the pelvis from 3 nerve roots. It passes through both greater and lesser sciatic foramina to enter the perineal region. The perineal branches pass anteromedially and divide into posterior scrotal branches. The posterior scrotal branches pass superficially to supply the skin and fascia of the perineum. It cross communicates with the inferior rectal nerve.

Next question

Question 545 of 560

The transversalis fascia contributes to which of the following?

Pectineal ligament

Deep inguinal ring

Cremaster muscle and fascia

Inguinal ligament

External spermatic fascia

The internal spermatic fascia (derived from transversalis fascia) invests: Ducuts deferens Testicular vessels

The principal outpouching of the transversalis fascia is the internal spermatic fascia. The mouth of the outpouching is the deep inguinal ring. Please rate this question:

Discuss and give feedback Next question Abdominal wall

The 2 main muscles of the abdominal wall are the rectus abdominis (anterior) and the quadratus lumborum (posterior). The remaining abdominal wall consists of 3 muscular layers. Each muscle passes from the lateral aspect of the quadratus lumborum posteriorly to the lateral margin of the rectus sheath anteriorly. Each layer is muscular posterolaterally and aponeurotic anteriorly.

Image sourced from Wikipedia

Muscles of abdominal wall External  Lies most superficially oblique  Originates from 5th to 12th ribs  Inserts into the anterior half of the outer aspect of the iliac crest, linea alba and pubic tubercle  More medially and superiorly to the arcuate line, the aponeurotic layer overlaps the rectus abdominis muscle  The lower border forms the inguinal ligament  The triangular expansion of the medial end of the inguinal ligament is the lacunar ligament.

Internal  Arises from the thoracolumbar fascia, the anterior 2/3 of the iliac crest oblique and the lateral 2/3 of the inguinal ligament  The muscle sweeps upwards to insert into the cartilages of the lower 3 ribs  The lower fibres form an aponeurosis that runs from the tenth costal cartilage to the body of the pubis  At its lowermost aspect it joins the fibres of the aponeurosis of transversus abdominis to form the conjoint tendon.

Transversus  Innermost muscle abdominis  Arises from the inner aspect of the costal cartilages of the lower 6 ribs , from the anterior 2/3 of the iliac crest and lateral 1/3 of the inguinal ligament  Its fibres run horizontally around the abdominal wall ending in an aponeurosis. The upper part runs posterior to the rectus abdominis. Lower down the fibres run anteriorly only.  The rectus abdominis lies medially; running from the pubic crest and symphysis to insert into the xiphoid process and 5th, 6th and 7th costal cartilages. The muscles lies in a aponeurosis as described above.  Nerve supply: anterior primary rami of T7-12

Surgical notes During abdominal surgery it is usually necessary to divide either the muscles or their aponeuroses. During a midline laparotomy it is desirable to divide the aponeurosis. This will leave the rectus sheath intact above the arcuate line and the muscles intact below it. Straying off the midline will often lead to damage to the rectus muscles, particularly below the arcuate line where they may often be in close proximity to each other. Next question

Question 546 of 560

A 63 year old man is undergoing a right pneumonectomy for carcinoma of the bronchus. As the surgeons approach the root of the lung, which structure will lie most posteriorly (in the anatomical plane)?

Phrenic nerve

Main bronchus

Vagus nerve

Pulmonary vein

Pulmonary artery

The vagus nerve is the most posteriorly located structure at the lung root. The phrenic nerve lies most anteriorly. Please rate this question:

Discuss and give feedback Next question Lung anatomy

The right lung is composed of 3 lobes divided by the oblique and transverse fissures. The left lung has two lobes divided by the oblique fissure.The apex of both lungs is approximately 4cm superior to the sterno-costal joint of the first rib. Immediately below this is a sulcus created by the subclavian artery.

Peripheral contact points of the lung

 Base: diaphragm  Costal surface: corresponds to the cavity of the chest  Mediastinal surface: Contacts the mediastinal pleura. Has the cardiac impression. Above and behind this concavity is a triangular depression named the hilum, where the structures which form the root of the lung enter and leave the viscus. These structures are invested by pleura, which, below the hilum and behind the pericardial impression, forms the pulmonary ligament

Right lung Above the hilum is the azygos vein; Superior to this is the groove for the superior vena cava and right innominate vein; behind this, and nearer the apex, is a furrow for the innominate artery. Behind the hilum and the attachment of the pulmonary ligament is a vertical groove for the oesophagus; In front and to the right of the lower part of the oesophageal groove is a deep concavity for the extrapericardiac portion of the inferior vena cava.

The root of the right lung lies behind the superior vena cava and the right atrium, and below the azygos vein.

The right main bronchus is shorter, wider and more vertical than the left main bronchus and therefore the route taken by most foreign bodies.

Image sourced from Wikipedia

Left lung Above the hilum is the furrow produced by the aortic arch, and then superiorly the groove accommodating the left subclavian artery; Behind the hilum and pulmonary ligament is a vertical groove produced by the descending aorta, and in front of this, near the base of the lung, is the lower part of the oesophagus.

The root of the left lung passes under the aortic arch and in front of the descending aorta.

Image sourced from Wikipedia

Inferior borders of both lungs

 6th rib in mid clavicular line  8th rib in mid axillary line  10th rib posteriorly

The pleura runs two ribs lower than the corresponding lung level.

Bronchopulmonary segments Segment number Right lung Left lung

1 Apical Apical

2 Posterior Posterior

3 Anterior Anterior

4 Lateral Superior lingular

5 Medial Inferior lingular

6 Superior (apical) Superior (apical) Segment number Right lung Left lung

7 Medial basal Medial basal

8 Anterior basal Anterior basal

9 Lateral basal Lateral basal

10 Posterior basal Posterior basal

Next question

Question 547 of 560

A 43 year old lady is undergoing an axillary node clearance for breast cancer. The nodal disease is bulky. During clearance of the level 3 nodes there is suddenly brisk haemorrhage. The most likely vessel responsible is:

Thoracoacromial artery

Cephalic vein

Thoracodorsal trunk

Internal mammary artery

Posterior circumflex humeral artery

The thoracoacromial artery pierces the pectoralis major and gives off branches within this space. The level 3 axillary nodes lie between pectoralis major and minor.Although the thoracodorsal trunk may be injured during an axillary dissection it does not lie within the level 3 nodes.

Please rate this question:

Discuss and give feedback

Next question

Thoracoacromial artery

The thoracoacromial artery (acromiothoracic artery; thoracic axis) is a short trunk, which arises from the forepart of the axillary artery, its origin being generally overlapped by the upper edge of the Pectoralis minor.

Projecting forward to the upper border of the Pectoralis minor, it pierces the coracoclavicular fascia and divides into four branches: pectoral, acromial, clavicular, and deltoid.

Branch Description

Pectoral Descends between the two Pectoral muscles, and is distributed to them and to the breast, branch anastomosing with the intercostal branches of the internal thoracic artery and with the lateral thoracic.

Acromial Runs laterally over the coracoid process and under the Deltoid, to which it gives branches; it branch then pierces that muscle and ends on the acromion in an arterial network formed by branches from the suprascapular, thoracoacromial, and posterior humeral circumflex arteries.

Clavicular Runs upwards and medially to the sternoclavicular joint, supplying this articulation, and the branch Subclavius.

Deltoid Arising with the acromial, it crosses over the Pectoralis minor and passes in the same branch groove as the cephalic vein, between the Pectoralis major and Deltoid, and gives branches to both muscles.

Next question

Question 548 of 560

A 73 year old lady with long standing atrial fibrillation develops a cold and pulseless white arm. A brachial embolus is suspected and a brachial embolectomy is performed. Which of the following structures is at greatest risk of injury during this procedure?

Radial nerve

Cephalic vein

Ulnar nerve

Median nerve

None of the above

The median nerve lies close to the brachial artery in the antecubital fossa. This is the usual site of surgical access to the brachial artery for an embolectomy procedure. The median nerve may be damaged during clumsy application of vascular clamps to the artery.

Please rate this question:

Discuss and give feedback

Next question

Brachial artery

The brachial artery begins at the lower border of teres major as a continuation of the axillary artery. It terminates in the cubital fossa at the level of the neck of the radius by dividing into the radial and ulnar arteries.

Relations Posterior relations include the long head of triceps with the radial nerve and profunda vessels intervening. Anteriorly it is overlapped by the medial border of biceps. It is crossed by the median nerve in the middle of the arm. In the cubital fossa it is separated from the median cubital vein by the bicipital aponeurosis. The basilic vein is in contact at the most proximal aspect of the cubital fossa and lies medially.

Next question

Question 549 of 560

A 73 year old lady is admitted with right iliac fossa pain. A plain abdominal x-ray is taken and the caecal diameter measured. Which of the following caecal diameters is pathological?

4cm

5cm

6cm

7cm

10cm

8 cm is still within normal limits. However, caecal diameters of 9 and 10 are pathological and should prompt further investigation. Please rate this question:

Discuss and give feedback Next question Right colon

Ileocaecal valve

 Entry point of the terminal ileum to the caecum  An important colonoscopic landmark  The ileocaecal valve is not always competent and this may allow partial decompression of an obstructed colon

Appendix

 At the base of the caecum the taenia coalesce to mark the base of the appendix  This is a reliable way of locating the appendix surgically and is a constant landmark  The appendix has a small mesentery (the mesoappendix) and in this runs the appendiceal artery, a branch of the ileocolic artery.

The posterior aspect of the right colon is extra peritoneal and the anterior aspect intraperitoneal.

Relations

 Posterior

Iliacus, Iliolumbar ligament, Quadratus lumborum, Transverse abdominis, Diaphragm at the tip of the last rib; Lateral cutaneous, ilioinguinal, and iliohypogastric nerves; the iliac branches of the iliolumbar vessels, the fourth lumbar artery, gonadal vessels, ureter and the right kidney.

 Superior

Right kidney which is embedded in the perinephric fat

 Medial

Mesentery which contains the ileocolic artery that supplies the right colon and terminal ileum. A further branch , the right colic artery, also contributes to supply the hepatic flexure and proximal transverse colon. Medially these pass through the mesentery to join the SMA. This occurs near to the head of the pancreas and care has to be taken when ligating the ileocolic artery near to its origin in cancer cases for fear of impinging on the SMA.

- Anterior Coils of small intestine, the right edge of the greater omentum, and the anterior abdominal wall.

Nerve supply

 Parasympathetic fibres of the vagus nerve (CN X)

Arterial supply

 Ileocolic artery and right colic artery, both branches of the SMA. While the ileocolic artery is almost always present, the right colic can be absent in 5-15% of individuals.

Next question

Question 550 of 560

Which of the following fingers is not a point of attachment for the palmar interossei?

Middle finger

Little finger

Ring finger

Index finger

None of the above

The middle finger has no attachment of the palmar interosseous.

Image sourced from Wikipedia

Please rate this question:

Discuss and give feedback Next question Hand

Anatomy of the hand Bones  8 Carpal bones  5 Metacarpals  14 phalanges

Intrinsic Muscles 7 Interossei - Supplied by ulnar nerve

 3 palmar-adduct fingers  4 dorsal- abduct fingers

Intrinsic muscles Lumbricals

 Flex MCPJ and extend the IPJ.  Origin deep flexor tendon and insertion dorsal extensor hood mechanism.  Innervation: 1st and 2nd- median nerve, 3rd and 4th- deep branch of the ulnar nerve.

Thenar eminence  Abductor pollicis brevis  Opponens pollicis  Flexor pollicis brevis

Hypothenar  Opponens digiti minimi eminence  Flexor digiti minimi brevis  Abductor digiti minimi

Image sourced from Wikipedia

Fascia and compartments of the palm The fascia of the palm is continuous with the antebrachial fascia and the fascia of the dorsum of the hand. The palmar fascia is thin over the thenar and hypothenar eminences. In contrast the palmar fascia is relatively thick. The palmar aponeurosis covers the soft tissues and overlies the flexor tendons. The apex of the palmar aponeurosis is continuous with the flexor retinaculum and the palmaris longus tendon. Distally, it forms four longitudinal digital bands that attach to the bases of the proximal phalanges, blending with the fibrous digital sheaths. A medial fibrous septum extends deeply from the medial border of the palmar aponeurosis to the 5th metacarpal. Lying medial to this are the hypothenar muscles. In a similar fashion, a lateral fibrous septum extends deeply from the lateral border of the palmar aponeurosis to the 3rd metacarpal. The thenar compartment lies lateral to this area. Lying between the thenar and hypothenar compartments is the central compartment. It contains the flexor tendons and their sheaths, the lumbricals, the superficial palmar arterial arch and the digital vessels and nerves. The deepest muscular plane is the adductor compartment, which contains adductor pollicis.

Short muscles of the hand These comprise the lumbricals and interossei. The four slender lumbrical muscles flex the fingers at the metacarpophalangeal joints and extend the interphalangeal joint. The four dorsal interossei are located between the metacarpals and the four palmar interossei lie on the palmar surface of the metacarpals in the interosseous compartment of the hand.

Long flexor tendons and sheaths in the hand The tendons of FDS and FDP enter the common flexor sheath deep to the flexor retinaculum. The tendons enter the central compartment of the hand and fan out to their respective digital synovial sheaths. Near the base of the proximal phalanx, the tendon of FDS splits to permit the passage of FDP. The FDP tendons are attached to the margins of the anterior aspect of the base of the distal phalanx. The fibrous digital sheaths contain the flexor tendons and their synovial sheaths. These extend from the heads of the metacarpals to the base of the distal phalanges. Next question

Question 551 of 560

A 33 year old man sustains an injury to his forearm and wrist. When examined in clinic he is unable to adduct his thumb. What is the most likely underlying nerve lesion?

Radial nerve

Superficial branch of the ulnar nerve

Median nerve

Posterior interosseous nerve

Deep branch of the ulnar nerve

Theme from April 2013 Exam Theme from April 2014 Exam Damage to the deep branch of the ulnar nerve may result in an inability to adduct the thumb. This is tested clinically by trying to withdraw a piece of paper from a patients hand grasped between thumb and index finger.

Please rate this question:

Discuss and give feedback

Next question

Adductor pollicis

Nerve Origin Insertion supply Actions Nerve Origin Insertion supply Actions

Tendon sheath of flexor Fibres of the two heads converge Deep branch Adducts the thumb carpi radialis on insertion into the ulnar aspect of the ulnar into the plane of the Bases of second, third and of the base of the proximal (C8, T1) palm and draws it to fourth metacarpals phalanx of the thumb the midline Anterior aspect of the trapezoid and capitate bones Transverse head comes from the longitudinal ride of the third metacarpal

Next question

Question 552 of 560

A 6 year old sustains a supracondylar fracture of the distal humerus. There are concerns that the radial nerve may have been injured. What is the relationship of the radial nerve to the humerus at this point?

Anterolateral

Anteromedial

Posterolateral

Posteromedial

Immediately anterior

The radial nerve lies anterolateral to the humerus in the supracondylar area. Please rate this question:

Discuss and give feedback Next question Radial nerve

Continuation of posterior cord of the brachial plexus (root values C5 to T1)

Path

 In the axilla: lies posterior to the axillary artery on subscapularis, latissimus dorsi and teres major.  Enters the arm between the brachial artery and the long head of triceps (medial to humerus).  Spirals around the posterior surface of the humerus in the groove for the radial nerve.  At the distal third of the lateral border of the humerus it then pierces the intermuscular septum and descends in front of the lateral epicondyle.  At the lateral epicondyle it lies deeply between brachialis and brachioradialis where it then divides into a superficial and deep terminal branch.  Deep branch crosses the supinator to become the posterior interosseous nerve.

In the image below the relationships of the radial nerve can be appreciated

Image sourced from Wikipedia

Regions innervated

 Triceps  Anconeus Motor (main nerve)  Brachioradialis  Extensor carpi radialis

 Supinator  Extensor carpi ulnaris  Extensor digitorum Motor (posterior  Extensor indicis interosseous branch)  Extensor digiti minimi  Extensor pollicis longus and brevis  Abductor pollicis longus

The area of skin supplying the proximal phalanges on the dorsal aspect of the Sensory hand is supplied by the radial nerve (this does not apply to the little finger and part of the ring finger)

Muscular innervation and effect of denervation Anatomical location Muscle affected Effect of paralysis Anatomical location Muscle affected Effect of paralysis

Shoulder Long head of triceps Minor effects on shoulder stability in abduction

Arm Triceps Loss of elbow extension

Forearm Supinator Weakening of supination of prone hand and Brachioradialis elbow flexion in mid prone position Extensor carpi radialis longus and brevis

The cutaneous sensation of the upper limb- illustrating the contribution of the radial nerve

Image sourced from Wikipedia

Next question

Question 553 of 560

Which of the following muscles is penetrated by the parotid duct?

Medial pterygoid

Buccinator

Levator anguli oris

Temporalis

Masseter

The duct crosses the masseter muscle and buccal fat pad and then penetrates the buccinator muscle to enter the oral cavity opposite the second upper molar tooth. Please rate this question:

Discuss and give feedback Next question Parotid gland

Anatomy of the parotid gland Location Overlying the mandibular ramus; anterior and inferior to the ear.

Salivary duct Crosses the masseter, pierces the buccinator and drains adjacent to the 2nd upper molar tooth (Stensen's duct).

Structures passing  Facial nerve (Mnemonic: The Zebra Buggered My Cat; Temporal through the gland Zygomatic, Buccal, Mandibular, Cervical)  External carotid artery  Retromandibular vein  Auriculotemporal nerve Relations  Anterior: masseter, medial pterygoid, superficial temporal and maxillary artery, facial nerve, stylomandibular ligament  Posterior: posterior belly digastric muscle, sternocleidomastoid, stylohyoid, internal carotid artery, mastoid process, styloid process

Arterial supply Branches of external carotid artery

Venous drainage Retromandibular vein

Lymphatic drainage Deep cervical nodes

Nerve innervation  Parasympathetic-Secretomotor  Sympathetic-Superior cervical ganglion  Sensory- Greater auricular nerve

Parasympathetic stimulation produces a water rich, serous saliva. Sympathetic stimulation leads to the production of a low volume, enzyme-rich saliva. Next question

Question 554 of 560

The following are true of the ulnar nerve except:

It innervates the palmar interossei

Derived from the medial cord of the brachial plexus

Supplies the muscles of the thenar eminence

Supplies the medial half of flexor digitorum profundus

Passes superficial to the flexor retinaculum

The muscles of the thenar eminence are supplied by the median nerve and atrophy of these is a feature of carpal tunnel syndrome. Please rate this question:

Discuss and give feedback Next question Ulnar nerve

Origin

 C8, T1

Supplies (no muscles in the upper arm)

 Flexor carpi ulnaris  Flexor digitorum profundus  Flexor digiti minimi  Abductor digiti minimi  Opponens digiti minimi  Adductor pollicis  Interossei muscle  Third and fourth lumbricals  Palmaris brevis

Path

 Posteromedial aspect of upper arm to flexor compartment of forearm, then along the ulnar. Passes beneath the flexor carpi ulnaris muscle, then superficially through the flexor retinaculum into the palm of the hand.

Image sourced from Wikipedia

Branches Branch Supplies

Muscular branch Flexor carpi ulnaris Medial half of the flexor digitorum profundus Branch Supplies

Palmar cutaneous branch (Arises near the Skin on the medial part of the palm middle of the forearm)

Dorsal cutaneous branch Dorsal surface of the medial part of the hand

Superficial branch Cutaneous fibres to the anterior surfaces of the medial one and one-half digits

Deep branch Hypothenar muscles All the interosseous muscles Third and fourth lumbricals Adductor pollicis Medial head of the flexor pollicis brevis

Effects of injury Damage at the wrist  Wasting and paralysis of intrinsic hand muscles (claw hand)  Wasting and paralysis of hypothenar muscles  Loss of sensation medial 1 and half fingers

Damage at the elbow  Radial deviation of the wrist  Clawing less in 4th and 5th digits

Next question

Question 555 of 560

How many valves lie between the superior vena cava and the right atrium?

None

One

Two

Three

Four

There are no valves which is why it is relatively easy to insert a CVP line from the internal jugular vein into the right atrium. Please rate this question:

Discuss and give feedback Next question Superior vena cava

Drainage

 Head and neck  Upper limbs  Thorax  Part of abdominal walls

Formation

 Subclavian and internal jugular veins unite to form the right and left brachiocephalic veins  These unite to form the SVC  Azygos vein joins the SVC before it enters the right atrium

Relations Anterior Anterior margins of the right lung and pleura

Posteromedial Trachea and right vagus nerve

Posterolateral Posterior aspects of right lung and pleura Pulmonary hilum is posterior

Right lateral Right phrenic nerve and pleura

Left lateral Brachiocephalic artery and ascending aorta

Developmental variations Anomalies of the connection of the SVC are recognised. In some individuals a persistent left sided SVC drains into the right atrium via an enlarged orifice of the coronary sinus. More rarely the left sided vena cava may connect directly with the superior aspect of the left atrium, usually associated with an un-roofing of the coronary sinus. The commonest lesion of the IVC is for its abdominal course to be interrupted, with drainage achieved via the azygos venous system. This may occur in patients with left sided atrial isomerism. Next question

Question 556 of 560

Which of the following options in relation to the liver is true?

Ligamentum venosum is an anterior relation of the liver

The portal triad comprises the hepatic artery, hepatic vein and tributary of the bile duct

The liver is completely covered by peritoneum

There are no nerves within the porta hepatis

The caudate lobe is superior to the porta hepatis

'VC goes with VC'

The ligamentun Venosum and Caudate is on same side as Vena Cava [posterior].

Ligamentum venosum is posterior to the liver. The portal triad contains the portal vein rather than the hepatic vein. There is the 'bare area of the liver' created by a void due to the layers being widely separated. There are sympathetic and parasympathetic nerves in the porta hepatis. Please rate this question:

Discuss and give feedback Next question Liver

Structure of the liver Right lobe  Supplied by right hepatic artery  Contains Couinaud segments V to VIII (-/+Sg I)

Left lobe  Supplied by the left hepatic artery  Contains Couinaud segments II to IV (+/- Sg1)

Quadrate lobe  Part of the right lobe anatomically, functionally is part of the left  Couinaud segment IV  Porta hepatis lies behind  On the right lies the gallbladder fossa  On the left lies the fossa for the umbilical vein

Caudate lobe  Supplied by both right and left hepatic arteries  Couinaud segment I  Lies behind the plane of the porta hepatis  Anterior and lateral to the inferior vena cava  Bile from the caudate lobe drains into both right and left hepatic ducts

Detailed knowledge of Couinaud segments is not required for MRCS

 Between the liver lobules are portal canals which contain the portal triad: Hepatic Artery, Portal Vein, tributary of Bile Duct.

Relations of the liver Anterior Postero inferiorly

Diaphragm Oesophagus

Xiphoid process Stomach

Duodenum

Hepatic flexure of colon

Right kidney

Gallbladder

Inferior vena cava

Porta hepatis Location Postero inferior surface, it joins nearly at right angles with the left sagittal fossa, and separates the caudate lobe behind from the quadrate lobe in front

Transmits  Common hepatic duct  Hepatic artery  Portal vein  Sympathetic and parasympathetic nerve fibres  Lymphatic drainage of the liver (and nodes)

Ligaments Falciform ligament  2 layer fold peritoneum from the umbilicus to anterior liver surface  Contains ligamentum teres (remnant umbilical vein)  On superior liver surface it splits into the coronary and left triangular ligaments

Ligamentum teres Joins the left branch of the portal vein in the porta hepatis

Ligamentum Remnant of ductus venosus venosum

Arterial supply

 Hepatic artery

Venous

 Hepatic veins  Portal vein

Nervous supply

 Sympathetic and parasympathetic trunks of coeliac plexus

Next question

Question 557 of 560

Which of the following structures does not pass anterior to the lateral malleolus?

Anterior tibial artery

Extensor digitorum longus

Tibialis anterior

Peroneus brevis

Peroneus tertius

Peroneus brevis passes posterior to the lateral malleolus. Please rate this question:

Discuss and give feedback Next question Lateral malleolus

Structures posterior to the lateral malleolus and superficial to superior peroneal retinaculum

 Sural nerve  Short saphenous vein

Structures posterior to the lateral malleolus and deep to superior peroneal retinaculum

 Peroneus longus tendon  Peroneus brevis tendon

The calcaneofibular ligament is attached at the lateral malleolus Next question

Question 558 of 560

The following statements regarding the rectus abdominis muscle are true except:

It runs from the symphysis pubis to the xiphoid process

Its nerve supply is from the ventral rami of the lower 6 thoracic nerves

It has collateral supply from both superior and inferior epigastric vessels

It lies in a muscular aponeurosis throughout its length

It has a number of tendinous intersections that penetrate through the anterior layer of the

muscle

Rectus abdominis

 Arises from the pubis.  Inserts into 5th, 6th, 7th costal cartilages.  The muscle lies in the rectal sheath, which also contains the superior and inferior epigastric artery and vein.  Action: flexion of thoracic and lumbar spine.  Nerve supply: anterior primary rami of T7-12.

The aponeurosis is deficient below the arcuate line. Please rate this question:

Discuss and give feedback Next question Abdominal wall

The 2 main muscles of the abdominal wall are the rectus abdominis (anterior) and the quadratus lumborum (posterior). The remaining abdominal wall consists of 3 muscular layers. Each muscle passes from the lateral aspect of the quadratus lumborum posteriorly to the lateral margin of the rectus sheath anteriorly. Each layer is muscular posterolaterally and aponeurotic anteriorly.

Image sourced from Wikipedia

Muscles of abdominal wall External  Lies most superficially oblique  Originates from 5th to 12th ribs  Inserts into the anterior half of the outer aspect of the iliac crest, linea alba and pubic tubercle  More medially and superiorly to the arcuate line, the aponeurotic layer overlaps the rectus abdominis muscle  The lower border forms the inguinal ligament  The triangular expansion of the medial end of the inguinal ligament is the lacunar ligament.

Internal  Arises from the thoracolumbar fascia, the anterior 2/3 of the iliac crest oblique and the lateral 2/3 of the inguinal ligament  The muscle sweeps upwards to insert into the cartilages of the lower 3 ribs  The lower fibres form an aponeurosis that runs from the tenth costal cartilage to the body of the pubis  At its lowermost aspect it joins the fibres of the aponeurosis of transversus abdominis to form the conjoint tendon.

Transversus  Innermost muscle abdominis  Arises from the inner aspect of the costal cartilages of the lower 6 ribs , from the anterior 2/3 of the iliac crest and lateral 1/3 of the inguinal ligament  Its fibres run horizontally around the abdominal wall ending in an aponeurosis. The upper part runs posterior to the rectus abdominis. Lower down the fibres run anteriorly only.  The rectus abdominis lies medially; running from the pubic crest and symphysis to insert into the xiphoid process and 5th, 6th and 7th costal cartilages. The muscles lies in a aponeurosis as described above.  Nerve supply: anterior primary rami of T7-12

Surgical notes During abdominal surgery it is usually necessary to divide either the muscles or their aponeuroses. During a midline laparotomy it is desirable to divide the aponeurosis. This will leave the rectus sheath intact above the arcuate line and the muscles intact below it. Straying off the midline will often lead to damage to the rectus muscles, particularly below the arcuate line where they may often be in close proximity to each other. Next question

Question 559 of 560

Which of the following statements relating to sternocleidomastoid is untrue?

The external jugular vein lies posteromedially.

It is supplied by the accessory nerve.

It has two heads of origin

It inserts into the lateral aspect of the mastoid process.

It marks the anterior border of the posterior triangle.

The external jugular vein lies lateral (i.e. superficial) to the sternocleidomastoid. Please rate this question:

Discuss and give feedback Next question Sternocleidomastoid

Anatomy Origin Rounded tendon attached to upper manubrium sterni and muscular head attached to medial third of the clavicle

Insertion Mastoid process of the temporal bone and lateral area of the superior nuchal line of the occipital bone

Innervation Spinal part of accessory nerve and anterior rami of C2 and C3 (proprioception)

Action  Both: extend the head at atlanto-occipital joint and flex the cervical vertebral column. Accessory muscles of inspiration.  Single: lateral flexion of neck, rotates head so face looks upward to the opposite side

Sternocleidomastoid divides the anterior and posterior triangles of the neck. Next question

Question 560 of 560

During liver mobilisation for a cadaveric liver transplant the hepatic ligaments will require mobilisation. Which of the following statements relating to these structures is untrue?

Lesser omentum arises from the porta hepatis and passes the lesser curvature of the

stomach

The falciform ligament divides into the left triangular ligament and coronary ligament

The liver has an area devoid of peritoneum

The coronary ligament is attached to the liver

The right triangular ligament is an early branch of the left triangular ligament

The right triangular ligament is a continuation of the coronary ligament. Please rate this question:

Discuss and give feedback

Liver

Structure of the liver Right lobe  Supplied by right hepatic artery  Contains Couinaud segments V to VIII (-/+Sg I)

Left lobe  Supplied by the left hepatic artery  Contains Couinaud segments II to IV (+/- Sg1)

Quadrate lobe  Part of the right lobe anatomically, functionally is part of the left  Couinaud segment IV  Porta hepatis lies behind  On the right lies the gallbladder fossa  On the left lies the fossa for the umbilical vein Caudate lobe  Supplied by both right and left hepatic arteries  Couinaud segment I  Lies behind the plane of the porta hepatis  Anterior and lateral to the inferior vena cava  Bile from the caudate lobe drains into both right and left hepatic ducts

Detailed knowledge of Couinaud segments is not required for MRCS

 Between the liver lobules are portal canals which contain the portal triad: Hepatic Artery, Portal Vein, tributary of Bile Duct.

Relations of the liver Anterior Postero inferiorly

Diaphragm Oesophagus

Xiphoid process Stomach

Duodenum

Hepatic flexure of colon

Right kidney

Gallbladder

Inferior vena cava

Porta hepatis Location Postero inferior surface, it joins nearly at right angles with the left sagittal fossa, and separates the caudate lobe behind from the quadrate lobe in front Transmits  Common hepatic duct  Hepatic artery  Portal vein  Sympathetic and parasympathetic nerve fibres  Lymphatic drainage of the liver (and nodes)

Ligaments Falciform ligament  2 layer fold peritoneum from the umbilicus to anterior liver surface  Contains ligamentum teres (remnant umbilical vein)  On superior liver surface it splits into the coronary and left triangular ligaments

Ligamentum teres Joins the left branch of the portal vein in the porta hepatis

Ligamentum Remnant of ductus venosus venosum

Arterial supply

 Hepatic artery

Venous

 Hepatic veins  Portal vein

Nervous supply

 Sympathetic and parasympathetic trunks of coeliac plexus